Olivia McRitchie
Quiz von , erstellt am more than 1 year ago

The fake patient I've used in this exemplar is a 44 year old osteoarthritis patient who works as an attorney and is married with 2 young children. The osteoarthritis is in his left knee. He is slightly overweight (10 pounds at most), but he keeps himself in very good health. He first approaches you wanting to know what this disorder could entail, then he asks about potential treatments. You give him teaching about nonpharmacological treatments, then send him on his way.

18
0
0
Olivia McRitchie
Erstellt von Olivia McRitchie vor mehr als 6 Jahre
Schließen

Exemplar 13.5: Osteoarthritis

Frage 1 von 11

1

You have a patient that has been newly diagnosed with osteoarthritis. You are educating her about her affliction. Select the INCORRECT statement about osteoarthritis.

Wähle eine der folgenden:

  • "In normal joints, you have something called articular cartilage covering the ends of your bones."

  • "In osteoarthritis, your cartilage has worn down, so now your bones are rubbing against each other."

  • "Small bone deposits called osteophytes may grow at the edges of the joint and change the shape of it."

  • "It is more likely to occur in your hips and knees than it is your spine and fingers."

  • "This disorder can either be secondary or idiopathic, with idiopathic meaning that doctors don't know what the cause is."

  • "This disorder can also be localized or generalized, with localized meaning that it occurs in 1 or 2 joints and generalized meaning 3 or more joints."

Erklärung

Frage 2 von 11

1

Select the individuals display a risk factor for osteoarthritis.

Wähle eine oder mehr der folgenden:

  • A black male male that is 60 years old.

  • A 50 year old Latino male that has worked in a Ford auto factory for the past 30 years.

  • A 45 year old white female that works as a teacher

  • A 50 year old, morbidly obese white female.

  • A 35 year old with Hashimoto's thyroiditis

  • A 50 year old Native American male with no pre-existing conditions

Erklärung

Frage 3 von 11

1

At any given age group, females are more likely to get osteoarthritis than men are

Wähle eins der folgenden:

  • WAHR
  • FALSCH

Erklärung

Frage 4 von 11

1

Select all the locations where men are more likely to get osteoarthritis.

Wähle eine oder mehr der folgenden:

  • Hips

  • Knees

  • Spine

  • Hands

  • Elbows

  • Toes

Erklärung

Frage 5 von 11

1

Select all the locations where women are more likely to get osteoarthritis,

Wähle eine oder mehr der folgenden:

  • Hip

  • Knees

  • Hands

  • Spine

  • Elbows

Erklärung

Frage 6 von 11

1

You are speaking to a 35 year old professional musician. She is multi-instrumental, but her main instrument is piano. Her mother was recently diagnosed with osteoarthritis, and she's concerned about developing it because of the nature of her career. She knows that if she develops osteoarthritis in her fingers and/or wrists, she'll have a hard time playing her instruments. Which of the following statements is RELEVANT, correct information to give this patient about preventing osteoarthritis?

Wähle eine der folgenden:

  • "Try to keep your weight within a normal range."

  • "Participate in a moderate exercise program that involves things like walking, jogging, cycling, or swimming."

  • "Make sure that you are using the correct finger positioning and posture while you are playing your instruments."

  • "Avoid engaging in repetitive wrist movement."

Erklärung

Frage 7 von 11

1

You have an elderly female patient who is exhibiting signs of a joint issue in her fingers and wrist. You are unsure what it is, but you are doing a head-to-toe assessment so you can collect information to give to he doctor. Select symptoms that may lead to you suggesting to the doctor that this patient has osteoarthritis.

Wähle eine oder mehr der folgenden:

  • Pain that is worsened by activity, but relieved by rest.

  • Pain and stiffness upon waking up.

  • Tenderness upon palpation of her fingers.

  • Swelling of her wrists.

  • Crackling when you ask her to move her wrists and curl her fingers.

  • Fatigue

  • Weakness

  • A symmetric pattern of joints affected.

  • Anorexia and weight loss.

  • Anemia

Erklärung

Frage 8 von 11

1

You have a 45 year old male patient, named John, that was recently diagnosed with osteoarthritis in his left knee. He works as an attorney, and has been with the same law firm for the past 10 years. He's very concerned because the disorder is already getting in the way of him playing with his young children. He wants you to tell him, with blunt honesty, what's going to end up happening to him. Select the accurate statements for teaching this patient complications that could arise with osteoarthritis.

Wähle eine oder mehr der folgenden:

  • "As this disorder progresses, it will increase your risk of falling."

  • "As this disorder progresses, you may have a hard time doing things like bathing yourself and dressing yourself."

  • "Because the treatment is expensive, you may suffer from financial strain."

  • "Because of your career, you should not have problems with being unable to work."

  • "Anxiety and depression may result because of the perpetual disability."

  • "Your marriage and relationship with your children will suffer if the mental illness develops."

Erklärung

Frage 9 von 11

1

After you tell John the potential complications of osteoarthritis, John asks you about potential treatments. Which of the following statements given to John is FALSE?

(In this one, we may be ignoring some aspects of therapeutic communication. Some of these may not seem like therapeutic communication, depending on how you read them).
(See description for details about John).

Wähle eine der folgenden:

  • "You can eundergo surgery, take medication, or participate in several non-pharmacological methods to treat this disorder."

  • "You can undergo arthroscopy, debridement, and joint irrigation, where the joint is visualized and small surgical instruments remove or trim painful structures."

  • "You can undergo a joint resurfacing, where a small amount of bone is removed and a metal replacement is fitted at the end of the bone."

  • "You can undergo an osteotomy, where a wedge of bone above or below the joint is removed to realign to joint and shift the weight away from the damaged area."

  • "You can undergo a joint fusion, where the surgeon permanently fuses 2 or more bones together."

  • "You can undergo an arthroplasy, where the damaged joint is totally removed and replaced."

  • "You're still young, so it seems unlikely that they'll perform an arthroplasty, but I can't be certain what surgery the doctor will want to perform. "

  • "Artificial joints from joint resurfacing and artheoplasty surgeries usually only about 5-10 years, so you'll need a replacement if you get one of those surgeries."

Erklärung

Frage 10 von 11

1

Weary about having to undergo surgery, John asks you about non-surgical and non-pharmacologic ways to relieve his osteoarthritis. You teach him about it, and are now assessing his knowledge. Select all the statements given by John that shows that he understands your instruction.

(See description for details about John).

Wähle eine oder mehr der folgenden:

  • "If I use heat to help my pain and increase my flexibility, I should not keep the hot pack on my knee for more than 30 minutes."

  • "If I use ice packs to help my pain and swelling, I should not keep it on my knee for longer than 30 minutes,"

  • "I can begin to wear a knee brace to keep my knee aligned."

  • "If I wear a knee brace, I can keep working at my normal pace."

  • "I should lose 10 pounds."

  • "I should get a cane."

  • "I should consult with a local physical therapist about a good exercise program for me."

  • "If I engage in an exercise program, I should push through any pain, as it is a natural response to exercising the affected knee."

Erklärung

Frage 11 von 11

1

You now have another patient who was recently diagnosed with osteoarthritis in her right ankle. She is a morbidly obese, white female that works as an actuarial scientist at an insurance company. She's complaining to you about how the disease is painful and causing her to slowly become disabled. She wants the progression to slow and she wants the pain to lessen, but she doesn't know what to do to stop it. Select the intervention that would be INCORRECT to suggest to this patient.

Wähle eine der folgenden:

  • Obtain the client's regular level of pain, on a scale to 0-10, with 0 being not bad at all and 10 being the worst pain imaginable.

  • Inform her about the proper dosage scheduling for over the counter ibuprofen.

  • Teach her how to safely apply heat and cold to her affected ankle.

  • Teach her to cycle rest and activity in her day to day life.

  • Assess her baseline range of motion in her affected ankle.

  • Refer the client to a physical therapist,

  • Inform the client that she may need a shower chair.

  • Gently suggest that she joins a program like Weight Watchers and looses some weight.

Erklärung